calculus limit question: another difficult limit problem











up vote
3
down vote

favorite
4












I have posted previously on a problem in a similar vein here:
Limit evaluation: very tough question, cannot use L'hopitals rule



I believe this problem is very similar, but it has stumped me.



$$lim_{x to 0}frac{1-frac12 x^2 - cosleft(frac{x}{1-x^2}right)}{x^4}$$



Really appreciate it if someone has some insight on this.This comes out to be indeterminate if one plugs in zero.
Following the idea from the link above, I tried to recognize this as derivative evaluated at zero of a function, BUT I could not find the function, because I tried to make this all over x, so that means the function I would create would generate a rational type with x^3 on the bottom.
I guess I should also try to look at some trig limit identities as well.



Hope someone out there can see how to navigate this problem.



P










share|cite|improve this question
























  • You should use Taylor formula to solve these kind of limits.
    – Emanuele Paolini
    Jan 3 '14 at 5:44










  • Is there a typo in your problem? Perhaps $$lim_{x to 0}frac{1-frac12x^2 - cos(frac{x}{1-x^2})}{x^4} ?$$
    – Stephen Montgomery-Smith
    Jan 3 '14 at 5:54












  • Yes, you are correct, I missed the 1/2 in front of x^2
    – Palu
    Jan 3 '14 at 5:56










  • Hi,I am using the taylor cosine series expansion.
    – Palu
    Jan 3 '14 at 5:57










  • @user99279: You should edit your question rather than leaving the correction in the comments.
    – user21820
    Jan 3 '14 at 5:58















up vote
3
down vote

favorite
4












I have posted previously on a problem in a similar vein here:
Limit evaluation: very tough question, cannot use L'hopitals rule



I believe this problem is very similar, but it has stumped me.



$$lim_{x to 0}frac{1-frac12 x^2 - cosleft(frac{x}{1-x^2}right)}{x^4}$$



Really appreciate it if someone has some insight on this.This comes out to be indeterminate if one plugs in zero.
Following the idea from the link above, I tried to recognize this as derivative evaluated at zero of a function, BUT I could not find the function, because I tried to make this all over x, so that means the function I would create would generate a rational type with x^3 on the bottom.
I guess I should also try to look at some trig limit identities as well.



Hope someone out there can see how to navigate this problem.



P










share|cite|improve this question
























  • You should use Taylor formula to solve these kind of limits.
    – Emanuele Paolini
    Jan 3 '14 at 5:44










  • Is there a typo in your problem? Perhaps $$lim_{x to 0}frac{1-frac12x^2 - cos(frac{x}{1-x^2})}{x^4} ?$$
    – Stephen Montgomery-Smith
    Jan 3 '14 at 5:54












  • Yes, you are correct, I missed the 1/2 in front of x^2
    – Palu
    Jan 3 '14 at 5:56










  • Hi,I am using the taylor cosine series expansion.
    – Palu
    Jan 3 '14 at 5:57










  • @user99279: You should edit your question rather than leaving the correction in the comments.
    – user21820
    Jan 3 '14 at 5:58













up vote
3
down vote

favorite
4









up vote
3
down vote

favorite
4






4





I have posted previously on a problem in a similar vein here:
Limit evaluation: very tough question, cannot use L'hopitals rule



I believe this problem is very similar, but it has stumped me.



$$lim_{x to 0}frac{1-frac12 x^2 - cosleft(frac{x}{1-x^2}right)}{x^4}$$



Really appreciate it if someone has some insight on this.This comes out to be indeterminate if one plugs in zero.
Following the idea from the link above, I tried to recognize this as derivative evaluated at zero of a function, BUT I could not find the function, because I tried to make this all over x, so that means the function I would create would generate a rational type with x^3 on the bottom.
I guess I should also try to look at some trig limit identities as well.



Hope someone out there can see how to navigate this problem.



P










share|cite|improve this question















I have posted previously on a problem in a similar vein here:
Limit evaluation: very tough question, cannot use L'hopitals rule



I believe this problem is very similar, but it has stumped me.



$$lim_{x to 0}frac{1-frac12 x^2 - cosleft(frac{x}{1-x^2}right)}{x^4}$$



Really appreciate it if someone has some insight on this.This comes out to be indeterminate if one plugs in zero.
Following the idea from the link above, I tried to recognize this as derivative evaluated at zero of a function, BUT I could not find the function, because I tried to make this all over x, so that means the function I would create would generate a rational type with x^3 on the bottom.
I guess I should also try to look at some trig limit identities as well.



Hope someone out there can see how to navigate this problem.



P







calculus limits






share|cite|improve this question















share|cite|improve this question













share|cite|improve this question




share|cite|improve this question








edited Nov 16 at 14:30









Paramanand Singh

48.4k555156




48.4k555156










asked Jan 3 '14 at 5:39









Palu

3252822




3252822












  • You should use Taylor formula to solve these kind of limits.
    – Emanuele Paolini
    Jan 3 '14 at 5:44










  • Is there a typo in your problem? Perhaps $$lim_{x to 0}frac{1-frac12x^2 - cos(frac{x}{1-x^2})}{x^4} ?$$
    – Stephen Montgomery-Smith
    Jan 3 '14 at 5:54












  • Yes, you are correct, I missed the 1/2 in front of x^2
    – Palu
    Jan 3 '14 at 5:56










  • Hi,I am using the taylor cosine series expansion.
    – Palu
    Jan 3 '14 at 5:57










  • @user99279: You should edit your question rather than leaving the correction in the comments.
    – user21820
    Jan 3 '14 at 5:58


















  • You should use Taylor formula to solve these kind of limits.
    – Emanuele Paolini
    Jan 3 '14 at 5:44










  • Is there a typo in your problem? Perhaps $$lim_{x to 0}frac{1-frac12x^2 - cos(frac{x}{1-x^2})}{x^4} ?$$
    – Stephen Montgomery-Smith
    Jan 3 '14 at 5:54












  • Yes, you are correct, I missed the 1/2 in front of x^2
    – Palu
    Jan 3 '14 at 5:56










  • Hi,I am using the taylor cosine series expansion.
    – Palu
    Jan 3 '14 at 5:57










  • @user99279: You should edit your question rather than leaving the correction in the comments.
    – user21820
    Jan 3 '14 at 5:58
















You should use Taylor formula to solve these kind of limits.
– Emanuele Paolini
Jan 3 '14 at 5:44




You should use Taylor formula to solve these kind of limits.
– Emanuele Paolini
Jan 3 '14 at 5:44












Is there a typo in your problem? Perhaps $$lim_{x to 0}frac{1-frac12x^2 - cos(frac{x}{1-x^2})}{x^4} ?$$
– Stephen Montgomery-Smith
Jan 3 '14 at 5:54






Is there a typo in your problem? Perhaps $$lim_{x to 0}frac{1-frac12x^2 - cos(frac{x}{1-x^2})}{x^4} ?$$
– Stephen Montgomery-Smith
Jan 3 '14 at 5:54














Yes, you are correct, I missed the 1/2 in front of x^2
– Palu
Jan 3 '14 at 5:56




Yes, you are correct, I missed the 1/2 in front of x^2
– Palu
Jan 3 '14 at 5:56












Hi,I am using the taylor cosine series expansion.
– Palu
Jan 3 '14 at 5:57




Hi,I am using the taylor cosine series expansion.
– Palu
Jan 3 '14 at 5:57












@user99279: You should edit your question rather than leaving the correction in the comments.
– user21820
Jan 3 '14 at 5:58




@user99279: You should edit your question rather than leaving the correction in the comments.
– user21820
Jan 3 '14 at 5:58










4 Answers
4






active

oldest

votes

















up vote
1
down vote



accepted










Like what Emanuele said, asymptotic expansions are useful for this kind of limits, and in fact better than L'Hopital (which fails miserably for some limits):



$frac{x}{1-x^2} in x + x^3 + O(x^5) to 0$ as $x to 0$



[We keep the error term so that at the end we know the error of the final approximation.]



$cos( frac{x}{1-x^2} ) in 1 - frac{1}{2} ( frac{x}{1-x^2} )^2 + frac{1}{24} ( frac{x}{1-x^2} )^4 + O( ( frac{x}{1-x^2} )^6 ) \
subset 1 - frac{1}{2} (x+x^3+O(x^5))^2 + frac{1}{24} (x+x^3+O(x^5))^4 + O(x^6) text{ as } x to 0 \
subset 1 - frac{1}{2} x^2 - frac{23}{24} x^4 + O(x^6) text{ as } x to 0$



[We can make the substitution into the Taylor expansion only because the input to $cos$ tends to 0.]



$frac{ 1 - frac{1}{2} x^2 - cos( frac{x}{1-x^2} ) }{ x^4 } in frac{ frac{23}{24} x^4 + O(x^6) }{ x^4 } = frac{23}{24} + O(x^2) to frac{23}{24}$ as $x to 0$



But you must make sure you understand the meaning of the Big-O notation and when and why they can be used. To make it more concrete, you can in many cases find explicit constants for bounds instead of using Big-O notation. For example:



$1 - frac{1}{2} x^2 + frac{1}{24} x^4 - frac{1}{720} x^6 le cos(x) le 1 - frac{1}{2} x^2 + frac{1}{24} x^4$ [obtained by repeated differentiation and Mean-value theorem]



$x + x^3 le frac{x}{1-x^2} le x + x^3 + 2 x^5$ for sufficiently small $x ge 0$



$x + x^3 ge frac{x}{1-x^2} ge x + x^3 + 2 x^5$ for sufficiently small $x le 0$






share|cite|improve this answer






























    up vote
    2
    down vote













    $frac 1{1-x^2}=1+x^2+O(x^4)$
    then $frac x{1-x^2}=x+x^3+O(x^5)$
    then $cosleft(frac x{1-x^2}right)=1-frac{x^2}{2}-x^4+frac{x^4}{24}+O(x^5)=1-frac{x^2}{2}-frac{23x^4}{24}+O(x^5)$ the limit is $frac{23}{24}$.






    share|cite|improve this answer




























      up vote
      0
      down vote













      Just expand $cosleft(frac{x}{1-x^2}right)$ using series expansion and simplify a bit






      share|cite|improve this answer























      • Ignore the terms which have the numerator greater than x^4 because that will become zero anyway
        – Aman
        May 30 '15 at 7:21


















      up vote
      0
      down vote













      Let $t=dfrac{x} {2(1-x^2)}$ to simplify typing. Note that $tto 0$ and $t/xto 1/2$ as $xto 0$. The numerator can be rewritten as $$2sin^2t-2cdotfrac{x^2}{4}=2left(sin t-frac {x} {2}right)left(sin t+frac{x}{2}right)=2ABtext{ (say)} $$ Clearly we have $$frac{B} {x} =frac{1}{2}+dfrac{sin t}{t}cdotfrac{t}{x}to frac {1}{2}+1cdotfrac{1}{2}=1$$ as $xto 0$. And $$frac{A} {x^3}=frac{sin t-t}{t^3}cdotfrac{t^3}{x^3}+frac{1}{2}cdotfrac{1}{1-x^2}to-frac{1}{6}cdotfrac{1}{8}+frac {1}{2}=frac{23}{48}$$ Therefore the given expression tends to $23/24$ as $xto 0$. In the above process we have used the standard limit $$lim_{tto 0}frac{sin t} {t} =1$$ and the limit $$lim_{tto 0}frac {sin t-t} {t^3}=-frac{1}{6}$$ which is easily proved via a single application of L'Hospital's Rule or via Taylor series for $sin t$.






      share|cite|improve this answer























        Your Answer





        StackExchange.ifUsing("editor", function () {
        return StackExchange.using("mathjaxEditing", function () {
        StackExchange.MarkdownEditor.creationCallbacks.add(function (editor, postfix) {
        StackExchange.mathjaxEditing.prepareWmdForMathJax(editor, postfix, [["$", "$"], ["\\(","\\)"]]);
        });
        });
        }, "mathjax-editing");

        StackExchange.ready(function() {
        var channelOptions = {
        tags: "".split(" "),
        id: "69"
        };
        initTagRenderer("".split(" "), "".split(" "), channelOptions);

        StackExchange.using("externalEditor", function() {
        // Have to fire editor after snippets, if snippets enabled
        if (StackExchange.settings.snippets.snippetsEnabled) {
        StackExchange.using("snippets", function() {
        createEditor();
        });
        }
        else {
        createEditor();
        }
        });

        function createEditor() {
        StackExchange.prepareEditor({
        heartbeatType: 'answer',
        convertImagesToLinks: true,
        noModals: true,
        showLowRepImageUploadWarning: true,
        reputationToPostImages: 10,
        bindNavPrevention: true,
        postfix: "",
        imageUploader: {
        brandingHtml: "Powered by u003ca class="icon-imgur-white" href="https://imgur.com/"u003eu003c/au003e",
        contentPolicyHtml: "User contributions licensed under u003ca href="https://creativecommons.org/licenses/by-sa/3.0/"u003ecc by-sa 3.0 with attribution requiredu003c/au003e u003ca href="https://stackoverflow.com/legal/content-policy"u003e(content policy)u003c/au003e",
        allowUrls: true
        },
        noCode: true, onDemand: true,
        discardSelector: ".discard-answer"
        ,immediatelyShowMarkdownHelp:true
        });


        }
        });














        draft saved

        draft discarded


















        StackExchange.ready(
        function () {
        StackExchange.openid.initPostLogin('.new-post-login', 'https%3a%2f%2fmath.stackexchange.com%2fquestions%2f625574%2fcalculus-limit-question-another-difficult-limit-problem%23new-answer', 'question_page');
        }
        );

        Post as a guest















        Required, but never shown

























        4 Answers
        4






        active

        oldest

        votes








        4 Answers
        4






        active

        oldest

        votes









        active

        oldest

        votes






        active

        oldest

        votes








        up vote
        1
        down vote



        accepted










        Like what Emanuele said, asymptotic expansions are useful for this kind of limits, and in fact better than L'Hopital (which fails miserably for some limits):



        $frac{x}{1-x^2} in x + x^3 + O(x^5) to 0$ as $x to 0$



        [We keep the error term so that at the end we know the error of the final approximation.]



        $cos( frac{x}{1-x^2} ) in 1 - frac{1}{2} ( frac{x}{1-x^2} )^2 + frac{1}{24} ( frac{x}{1-x^2} )^4 + O( ( frac{x}{1-x^2} )^6 ) \
        subset 1 - frac{1}{2} (x+x^3+O(x^5))^2 + frac{1}{24} (x+x^3+O(x^5))^4 + O(x^6) text{ as } x to 0 \
        subset 1 - frac{1}{2} x^2 - frac{23}{24} x^4 + O(x^6) text{ as } x to 0$



        [We can make the substitution into the Taylor expansion only because the input to $cos$ tends to 0.]



        $frac{ 1 - frac{1}{2} x^2 - cos( frac{x}{1-x^2} ) }{ x^4 } in frac{ frac{23}{24} x^4 + O(x^6) }{ x^4 } = frac{23}{24} + O(x^2) to frac{23}{24}$ as $x to 0$



        But you must make sure you understand the meaning of the Big-O notation and when and why they can be used. To make it more concrete, you can in many cases find explicit constants for bounds instead of using Big-O notation. For example:



        $1 - frac{1}{2} x^2 + frac{1}{24} x^4 - frac{1}{720} x^6 le cos(x) le 1 - frac{1}{2} x^2 + frac{1}{24} x^4$ [obtained by repeated differentiation and Mean-value theorem]



        $x + x^3 le frac{x}{1-x^2} le x + x^3 + 2 x^5$ for sufficiently small $x ge 0$



        $x + x^3 ge frac{x}{1-x^2} ge x + x^3 + 2 x^5$ for sufficiently small $x le 0$






        share|cite|improve this answer



























          up vote
          1
          down vote



          accepted










          Like what Emanuele said, asymptotic expansions are useful for this kind of limits, and in fact better than L'Hopital (which fails miserably for some limits):



          $frac{x}{1-x^2} in x + x^3 + O(x^5) to 0$ as $x to 0$



          [We keep the error term so that at the end we know the error of the final approximation.]



          $cos( frac{x}{1-x^2} ) in 1 - frac{1}{2} ( frac{x}{1-x^2} )^2 + frac{1}{24} ( frac{x}{1-x^2} )^4 + O( ( frac{x}{1-x^2} )^6 ) \
          subset 1 - frac{1}{2} (x+x^3+O(x^5))^2 + frac{1}{24} (x+x^3+O(x^5))^4 + O(x^6) text{ as } x to 0 \
          subset 1 - frac{1}{2} x^2 - frac{23}{24} x^4 + O(x^6) text{ as } x to 0$



          [We can make the substitution into the Taylor expansion only because the input to $cos$ tends to 0.]



          $frac{ 1 - frac{1}{2} x^2 - cos( frac{x}{1-x^2} ) }{ x^4 } in frac{ frac{23}{24} x^4 + O(x^6) }{ x^4 } = frac{23}{24} + O(x^2) to frac{23}{24}$ as $x to 0$



          But you must make sure you understand the meaning of the Big-O notation and when and why they can be used. To make it more concrete, you can in many cases find explicit constants for bounds instead of using Big-O notation. For example:



          $1 - frac{1}{2} x^2 + frac{1}{24} x^4 - frac{1}{720} x^6 le cos(x) le 1 - frac{1}{2} x^2 + frac{1}{24} x^4$ [obtained by repeated differentiation and Mean-value theorem]



          $x + x^3 le frac{x}{1-x^2} le x + x^3 + 2 x^5$ for sufficiently small $x ge 0$



          $x + x^3 ge frac{x}{1-x^2} ge x + x^3 + 2 x^5$ for sufficiently small $x le 0$






          share|cite|improve this answer

























            up vote
            1
            down vote



            accepted







            up vote
            1
            down vote



            accepted






            Like what Emanuele said, asymptotic expansions are useful for this kind of limits, and in fact better than L'Hopital (which fails miserably for some limits):



            $frac{x}{1-x^2} in x + x^3 + O(x^5) to 0$ as $x to 0$



            [We keep the error term so that at the end we know the error of the final approximation.]



            $cos( frac{x}{1-x^2} ) in 1 - frac{1}{2} ( frac{x}{1-x^2} )^2 + frac{1}{24} ( frac{x}{1-x^2} )^4 + O( ( frac{x}{1-x^2} )^6 ) \
            subset 1 - frac{1}{2} (x+x^3+O(x^5))^2 + frac{1}{24} (x+x^3+O(x^5))^4 + O(x^6) text{ as } x to 0 \
            subset 1 - frac{1}{2} x^2 - frac{23}{24} x^4 + O(x^6) text{ as } x to 0$



            [We can make the substitution into the Taylor expansion only because the input to $cos$ tends to 0.]



            $frac{ 1 - frac{1}{2} x^2 - cos( frac{x}{1-x^2} ) }{ x^4 } in frac{ frac{23}{24} x^4 + O(x^6) }{ x^4 } = frac{23}{24} + O(x^2) to frac{23}{24}$ as $x to 0$



            But you must make sure you understand the meaning of the Big-O notation and when and why they can be used. To make it more concrete, you can in many cases find explicit constants for bounds instead of using Big-O notation. For example:



            $1 - frac{1}{2} x^2 + frac{1}{24} x^4 - frac{1}{720} x^6 le cos(x) le 1 - frac{1}{2} x^2 + frac{1}{24} x^4$ [obtained by repeated differentiation and Mean-value theorem]



            $x + x^3 le frac{x}{1-x^2} le x + x^3 + 2 x^5$ for sufficiently small $x ge 0$



            $x + x^3 ge frac{x}{1-x^2} ge x + x^3 + 2 x^5$ for sufficiently small $x le 0$






            share|cite|improve this answer














            Like what Emanuele said, asymptotic expansions are useful for this kind of limits, and in fact better than L'Hopital (which fails miserably for some limits):



            $frac{x}{1-x^2} in x + x^3 + O(x^5) to 0$ as $x to 0$



            [We keep the error term so that at the end we know the error of the final approximation.]



            $cos( frac{x}{1-x^2} ) in 1 - frac{1}{2} ( frac{x}{1-x^2} )^2 + frac{1}{24} ( frac{x}{1-x^2} )^4 + O( ( frac{x}{1-x^2} )^6 ) \
            subset 1 - frac{1}{2} (x+x^3+O(x^5))^2 + frac{1}{24} (x+x^3+O(x^5))^4 + O(x^6) text{ as } x to 0 \
            subset 1 - frac{1}{2} x^2 - frac{23}{24} x^4 + O(x^6) text{ as } x to 0$



            [We can make the substitution into the Taylor expansion only because the input to $cos$ tends to 0.]



            $frac{ 1 - frac{1}{2} x^2 - cos( frac{x}{1-x^2} ) }{ x^4 } in frac{ frac{23}{24} x^4 + O(x^6) }{ x^4 } = frac{23}{24} + O(x^2) to frac{23}{24}$ as $x to 0$



            But you must make sure you understand the meaning of the Big-O notation and when and why they can be used. To make it more concrete, you can in many cases find explicit constants for bounds instead of using Big-O notation. For example:



            $1 - frac{1}{2} x^2 + frac{1}{24} x^4 - frac{1}{720} x^6 le cos(x) le 1 - frac{1}{2} x^2 + frac{1}{24} x^4$ [obtained by repeated differentiation and Mean-value theorem]



            $x + x^3 le frac{x}{1-x^2} le x + x^3 + 2 x^5$ for sufficiently small $x ge 0$



            $x + x^3 ge frac{x}{1-x^2} ge x + x^3 + 2 x^5$ for sufficiently small $x le 0$







            share|cite|improve this answer














            share|cite|improve this answer



            share|cite|improve this answer








            edited Jan 3 '14 at 6:29

























            answered Jan 3 '14 at 6:24









            user21820

            38.1k541150




            38.1k541150






















                up vote
                2
                down vote













                $frac 1{1-x^2}=1+x^2+O(x^4)$
                then $frac x{1-x^2}=x+x^3+O(x^5)$
                then $cosleft(frac x{1-x^2}right)=1-frac{x^2}{2}-x^4+frac{x^4}{24}+O(x^5)=1-frac{x^2}{2}-frac{23x^4}{24}+O(x^5)$ the limit is $frac{23}{24}$.






                share|cite|improve this answer

























                  up vote
                  2
                  down vote













                  $frac 1{1-x^2}=1+x^2+O(x^4)$
                  then $frac x{1-x^2}=x+x^3+O(x^5)$
                  then $cosleft(frac x{1-x^2}right)=1-frac{x^2}{2}-x^4+frac{x^4}{24}+O(x^5)=1-frac{x^2}{2}-frac{23x^4}{24}+O(x^5)$ the limit is $frac{23}{24}$.






                  share|cite|improve this answer























                    up vote
                    2
                    down vote










                    up vote
                    2
                    down vote









                    $frac 1{1-x^2}=1+x^2+O(x^4)$
                    then $frac x{1-x^2}=x+x^3+O(x^5)$
                    then $cosleft(frac x{1-x^2}right)=1-frac{x^2}{2}-x^4+frac{x^4}{24}+O(x^5)=1-frac{x^2}{2}-frac{23x^4}{24}+O(x^5)$ the limit is $frac{23}{24}$.






                    share|cite|improve this answer












                    $frac 1{1-x^2}=1+x^2+O(x^4)$
                    then $frac x{1-x^2}=x+x^3+O(x^5)$
                    then $cosleft(frac x{1-x^2}right)=1-frac{x^2}{2}-x^4+frac{x^4}{24}+O(x^5)=1-frac{x^2}{2}-frac{23x^4}{24}+O(x^5)$ the limit is $frac{23}{24}$.







                    share|cite|improve this answer












                    share|cite|improve this answer



                    share|cite|improve this answer










                    answered Jan 3 '14 at 6:19









                    Mohamed

                    2,864822




                    2,864822






















                        up vote
                        0
                        down vote













                        Just expand $cosleft(frac{x}{1-x^2}right)$ using series expansion and simplify a bit






                        share|cite|improve this answer























                        • Ignore the terms which have the numerator greater than x^4 because that will become zero anyway
                          – Aman
                          May 30 '15 at 7:21















                        up vote
                        0
                        down vote













                        Just expand $cosleft(frac{x}{1-x^2}right)$ using series expansion and simplify a bit






                        share|cite|improve this answer























                        • Ignore the terms which have the numerator greater than x^4 because that will become zero anyway
                          – Aman
                          May 30 '15 at 7:21













                        up vote
                        0
                        down vote










                        up vote
                        0
                        down vote









                        Just expand $cosleft(frac{x}{1-x^2}right)$ using series expansion and simplify a bit






                        share|cite|improve this answer














                        Just expand $cosleft(frac{x}{1-x^2}right)$ using series expansion and simplify a bit







                        share|cite|improve this answer














                        share|cite|improve this answer



                        share|cite|improve this answer








                        edited Jun 3 '15 at 16:04









                        user85503

                        4851415




                        4851415










                        answered May 30 '15 at 7:19









                        Aman

                        11




                        11












                        • Ignore the terms which have the numerator greater than x^4 because that will become zero anyway
                          – Aman
                          May 30 '15 at 7:21


















                        • Ignore the terms which have the numerator greater than x^4 because that will become zero anyway
                          – Aman
                          May 30 '15 at 7:21
















                        Ignore the terms which have the numerator greater than x^4 because that will become zero anyway
                        – Aman
                        May 30 '15 at 7:21




                        Ignore the terms which have the numerator greater than x^4 because that will become zero anyway
                        – Aman
                        May 30 '15 at 7:21










                        up vote
                        0
                        down vote













                        Let $t=dfrac{x} {2(1-x^2)}$ to simplify typing. Note that $tto 0$ and $t/xto 1/2$ as $xto 0$. The numerator can be rewritten as $$2sin^2t-2cdotfrac{x^2}{4}=2left(sin t-frac {x} {2}right)left(sin t+frac{x}{2}right)=2ABtext{ (say)} $$ Clearly we have $$frac{B} {x} =frac{1}{2}+dfrac{sin t}{t}cdotfrac{t}{x}to frac {1}{2}+1cdotfrac{1}{2}=1$$ as $xto 0$. And $$frac{A} {x^3}=frac{sin t-t}{t^3}cdotfrac{t^3}{x^3}+frac{1}{2}cdotfrac{1}{1-x^2}to-frac{1}{6}cdotfrac{1}{8}+frac {1}{2}=frac{23}{48}$$ Therefore the given expression tends to $23/24$ as $xto 0$. In the above process we have used the standard limit $$lim_{tto 0}frac{sin t} {t} =1$$ and the limit $$lim_{tto 0}frac {sin t-t} {t^3}=-frac{1}{6}$$ which is easily proved via a single application of L'Hospital's Rule or via Taylor series for $sin t$.






                        share|cite|improve this answer



























                          up vote
                          0
                          down vote













                          Let $t=dfrac{x} {2(1-x^2)}$ to simplify typing. Note that $tto 0$ and $t/xto 1/2$ as $xto 0$. The numerator can be rewritten as $$2sin^2t-2cdotfrac{x^2}{4}=2left(sin t-frac {x} {2}right)left(sin t+frac{x}{2}right)=2ABtext{ (say)} $$ Clearly we have $$frac{B} {x} =frac{1}{2}+dfrac{sin t}{t}cdotfrac{t}{x}to frac {1}{2}+1cdotfrac{1}{2}=1$$ as $xto 0$. And $$frac{A} {x^3}=frac{sin t-t}{t^3}cdotfrac{t^3}{x^3}+frac{1}{2}cdotfrac{1}{1-x^2}to-frac{1}{6}cdotfrac{1}{8}+frac {1}{2}=frac{23}{48}$$ Therefore the given expression tends to $23/24$ as $xto 0$. In the above process we have used the standard limit $$lim_{tto 0}frac{sin t} {t} =1$$ and the limit $$lim_{tto 0}frac {sin t-t} {t^3}=-frac{1}{6}$$ which is easily proved via a single application of L'Hospital's Rule or via Taylor series for $sin t$.






                          share|cite|improve this answer

























                            up vote
                            0
                            down vote










                            up vote
                            0
                            down vote









                            Let $t=dfrac{x} {2(1-x^2)}$ to simplify typing. Note that $tto 0$ and $t/xto 1/2$ as $xto 0$. The numerator can be rewritten as $$2sin^2t-2cdotfrac{x^2}{4}=2left(sin t-frac {x} {2}right)left(sin t+frac{x}{2}right)=2ABtext{ (say)} $$ Clearly we have $$frac{B} {x} =frac{1}{2}+dfrac{sin t}{t}cdotfrac{t}{x}to frac {1}{2}+1cdotfrac{1}{2}=1$$ as $xto 0$. And $$frac{A} {x^3}=frac{sin t-t}{t^3}cdotfrac{t^3}{x^3}+frac{1}{2}cdotfrac{1}{1-x^2}to-frac{1}{6}cdotfrac{1}{8}+frac {1}{2}=frac{23}{48}$$ Therefore the given expression tends to $23/24$ as $xto 0$. In the above process we have used the standard limit $$lim_{tto 0}frac{sin t} {t} =1$$ and the limit $$lim_{tto 0}frac {sin t-t} {t^3}=-frac{1}{6}$$ which is easily proved via a single application of L'Hospital's Rule or via Taylor series for $sin t$.






                            share|cite|improve this answer














                            Let $t=dfrac{x} {2(1-x^2)}$ to simplify typing. Note that $tto 0$ and $t/xto 1/2$ as $xto 0$. The numerator can be rewritten as $$2sin^2t-2cdotfrac{x^2}{4}=2left(sin t-frac {x} {2}right)left(sin t+frac{x}{2}right)=2ABtext{ (say)} $$ Clearly we have $$frac{B} {x} =frac{1}{2}+dfrac{sin t}{t}cdotfrac{t}{x}to frac {1}{2}+1cdotfrac{1}{2}=1$$ as $xto 0$. And $$frac{A} {x^3}=frac{sin t-t}{t^3}cdotfrac{t^3}{x^3}+frac{1}{2}cdotfrac{1}{1-x^2}to-frac{1}{6}cdotfrac{1}{8}+frac {1}{2}=frac{23}{48}$$ Therefore the given expression tends to $23/24$ as $xto 0$. In the above process we have used the standard limit $$lim_{tto 0}frac{sin t} {t} =1$$ and the limit $$lim_{tto 0}frac {sin t-t} {t^3}=-frac{1}{6}$$ which is easily proved via a single application of L'Hospital's Rule or via Taylor series for $sin t$.







                            share|cite|improve this answer














                            share|cite|improve this answer



                            share|cite|improve this answer








                            edited Nov 16 at 14:27

























                            answered Nov 16 at 14:19









                            Paramanand Singh

                            48.4k555156




                            48.4k555156






























                                draft saved

                                draft discarded




















































                                Thanks for contributing an answer to Mathematics Stack Exchange!


                                • Please be sure to answer the question. Provide details and share your research!

                                But avoid



                                • Asking for help, clarification, or responding to other answers.

                                • Making statements based on opinion; back them up with references or personal experience.


                                Use MathJax to format equations. MathJax reference.


                                To learn more, see our tips on writing great answers.





                                Some of your past answers have not been well-received, and you're in danger of being blocked from answering.


                                Please pay close attention to the following guidance:


                                • Please be sure to answer the question. Provide details and share your research!

                                But avoid



                                • Asking for help, clarification, or responding to other answers.

                                • Making statements based on opinion; back them up with references or personal experience.


                                To learn more, see our tips on writing great answers.




                                draft saved


                                draft discarded














                                StackExchange.ready(
                                function () {
                                StackExchange.openid.initPostLogin('.new-post-login', 'https%3a%2f%2fmath.stackexchange.com%2fquestions%2f625574%2fcalculus-limit-question-another-difficult-limit-problem%23new-answer', 'question_page');
                                }
                                );

                                Post as a guest















                                Required, but never shown





















































                                Required, but never shown














                                Required, but never shown












                                Required, but never shown







                                Required, but never shown

































                                Required, but never shown














                                Required, but never shown












                                Required, but never shown







                                Required, but never shown







                                Popular posts from this blog

                                Plaza Victoria

                                In PowerPoint, is there a keyboard shortcut for bulleted / numbered list?

                                How to put 3 figures in Latex with 2 figures side by side and 1 below these side by side images but in...